2
$\begingroup$

Let $(X, \cal{M},\mu)$ be a finite positive measure space and $f$ a $\mu$-a.e. strictly positive measurable function on $X$. If $E_n\in\mathcal{M}$, for $n=1,2,\ldots $ and $\displaystyle \lim_{n\rightarrow\infty} \int_{E_n}f d\mu=0$, prove that $\displaystyle\lim_{n\rightarrow\infty}\mu(E_n)=0$.

  • 3
    I'd just like to point out a good recent example of [how to ask a question the good way](http://math.stackexchange.com/questions/149870/proving-injectivity#149870). We feel much better answering questions to people who show in their question that they have thought about it.2012-05-26

3 Answers 3

1

Since $f$ is almost everywhere strictly positive, the increasing sequence of sets $A_n=\{x\in X:f(x)>1/n\}$ has the property that $\lim_{n\to\infty} \mu(A_n)=\mu(X).$ Now $\int_E f ~d\mu implies that $\mu(E\cap A_n)\cdot1/n and hence $\mu(E\cap A_n) for all $n$ and $r>0$.

So let $\epsilon>0$. Choose $n$ so that $\mu(X\backslash A_n)<\epsilon/2$. For $N$ large enough, $\int_{E_N}f~d\mu<\epsilon/(2n)$ and hence $\mu(E_N\cap A_n) Now $\mu(E_N)\leq\mu(E_N\cap A_n)+\mu(X\backslash A_n)<\epsilon/2+\epsilon/2=\epsilon.$

2

Hints:

  • If $f$ is strictly positive almost everywhere then for "most of $X$" (which you can make arbitrarily large) there is a positive value it is greater than.

  • So unless $E_n$ gets "small enough" permanently in the tail, you can show $\int_{E_n}f d\mu$ most be greater than a particular positive value infinitely often, by comparing the overlap between $E_n$ and "most of $X$".

  • You can formalise this, as with other limits, taking care with the relationship between "most" and "small enough".

1

Since the measure space is assumed to be finite, we assume it to be a probability space. It is useful to first divide $X$ into pairwise disjoint measurable subsets $A_0:=\{x\in X:f(x)\geq 1\}$ $ A_n:=\{x\in X:\tfrac{1}{n+1}\leq f(x)<\tfrac{1}{n}\}. $ Note that since $f$ is assumed to be positive $\mu$-a.e., we have that $\sum_{n=0}^\infty\mu(A_n)=1$. Using the sets $A_n$, we are able to get the integrals $\int_{E_n}f\,\mathrm{d}\mu$ into the story: $ \lim_{k\to\infty}\mu(E_k)=\lim_{k\to\infty}\sum_{n\in\mathbb{N}}\mu(E_k\cap A_n)\leq\lim_{k\to\infty}\sum_{n\in\mathbb{N}}\int_{E_k}\mathbf{1}_{A_n}(n+1)f\,\mathrm{d}\mu $ Now, wouldn't it be good if we can take the limit inside? Try to prove if that is possible. And then see if you can use the hypothesis. Hopefully I've got you going now :)